Proving the Shift Theorem in an Inverse Fourier Transform

Click For Summary
The discussion revolves around proving that the inverse Fourier transform of e^{i\omega \beta}F(\omega) results in f(x-\beta). The approach involves manipulating the integral of the inverse Fourier transform by substituting k = β - x. The integral is then expressed as f(k), which leads to the conclusion that f(k) equals f(β - x). A participant expresses confusion over a potential sign error in their calculations and seeks assistance in identifying the mistake. The conversation highlights the importance of careful sign management in Fourier transform proofs.
mjordan2nd
Messages
173
Reaction score
1

Homework Statement



We are asked to prove that if F(\omega ) is the Fourier transform of f(x) then prove that the inverse Fourier transform of e^{i\omega \beta}F(\omega) is f(x-\beta )

Homework Equations



F(\omega)=\frac{1}{2\pi}\int^{\infty}_{-\infty}f(x)e^{i\omega x}dx
f(x)=\int^{\infty}_{-\infty}F(\omega)e^{-i\omega x}d\omega

The Attempt at a Solution



We want to find the inverse Fourier transform of \int^{\infty}_{-\infty}F(\omega)e^{i\omega \beta}e^{-i\omega x}

Setting k=\beta - x

\int^{\infty}_{-\infty}F(\omega)e^{i\omega \beta -x}d\omega = \int^{\infty}_{-\infty}F(\omega)e^{i\omega k}d\omega=f(k)=f(\beta -x)

I seem to have the signs reversed in my answer. Could anyone help me spot where I missed a sign change please. Thanks.
 
Physics news on Phys.org
For some reason my edits aren't showing up...
 
Could anyone help me out?
 
Question: A clock's minute hand has length 4 and its hour hand has length 3. What is the distance between the tips at the moment when it is increasing most rapidly?(Putnam Exam Question) Answer: Making assumption that both the hands moves at constant angular velocities, the answer is ## \sqrt{7} .## But don't you think this assumption is somewhat doubtful and wrong?

Similar threads

Replies
1
Views
2K
  • · Replies 2 ·
Replies
2
Views
3K
Replies
2
Views
2K
  • · Replies 1 ·
Replies
1
Views
1K
  • · Replies 1 ·
Replies
1
Views
2K
  • · Replies 3 ·
Replies
3
Views
2K
  • · Replies 3 ·
Replies
3
Views
3K
Replies
6
Views
1K
  • · Replies 6 ·
Replies
6
Views
3K
  • · Replies 12 ·
Replies
12
Views
2K